LSAT and Law School Admissions Forum

Get expert LSAT preparation and law school admissions advice from PowerScore Test Preparation.

User avatar
 Dave Killoran
PowerScore Staff
  • PowerScore Staff
  • Posts: 5852
  • Joined: Mar 25, 2011
|
#43433
Complete Question Explanation
(The complete setup for this game can be found here: lsat/viewtopic.php?t=16399)

The correct answer choice is (B)

At first glance, this appears to be a difficult Global question. Remember, if you do not have a ready inference to apply to this type of question, prepare to use hypotheticals and previous information.

Answer choice (A) can be eliminated by the question stem to #21.

Answer choice (B) is the correct answer choice.

Answer choice (C) can be eliminated by the hypothetical produced in #21 (and in our discussion of what occurs when N is not selected).

Answer choice (D) can be eliminated by the hypothetical array we produced when discussing the selection of H, K, and N (when H, K, and N are selected, then any three of G, L, V, and Z can be selected, allowing for a hypothetical with only one monkey).

Answer choice (E) can be eliminated by the question stem to #21.

Get the most out of your LSAT Prep Plus subscription.

Analyze and track your performance with our Testing and Analytics Package.